Staffetta algebra

Polinomi, disuguaglianze, numeri complessi, ...
Avatar utente
jordan
Messaggi: 3988
Iscritto il: 02 feb 2007, 21:19
Località: Pescara
Contatta:

Messaggio da jordan »

Hint problema 6: le radici di p(x) hanno modulo $ \displaystyle 2^{\frac{2}{n}} $..
The only goal of science is the honor of the human spirit.
Thebear
Messaggi: 311
Iscritto il: 13 feb 2008, 16:23
Località: Torino

Messaggio da Thebear »

Non vorrei aver malinterpretato il significato di "polinomio irriducibile", ma se lo intendo come "non scomponibile" la soluzione segue direttamente dall'identità di Sophie Germain. cioè:

Immaginiamo che sia $ n=4k $ allora si avrà

$ p(x)=x^{4k}+4=(x^{2k}+2)^2-4x^{2k}= $

$ =(x^{2k}+2+2x^k)(x^{2k}+2-2x^k) $ che quindi è scomponibile.

Aspettando però che qualcuno mi dica che "irriducibile" è diverso da "non scomponibile" non posto nessun nuovo problema. :roll:
Edoardo
Avatar utente
mitchan88
Messaggi: 469
Iscritto il: 01 gen 1970, 01:00
Contatta:

Messaggio da mitchan88 »

Thebear ha scritto:Non vorrei aver malinterpretato il significato di "polinomio irriducibile", ma se lo intendo come "non scomponibile" la soluzione segue direttamente dall'identità di Sophie Germain. cioè:

Immaginiamo che sia $ n=4k $ allora si avrà

$ p(x)=x^{4k}+4=(x^{2k}+2)^2-4x^{2k}= $

$ =(x^{2k}+2+2x^k)(x^{2k}+2-2x^k) $ che quindi è scomponibile.

Aspettando però che qualcuno mi dica che "irriducibile" è diverso da "non scomponibile" non posto nessun nuovo problema. :roll:
Fin qui è corretto!
Ma ti manca da dimostrare che se n non è multiplo di 4 è effettivamente irriducibile...
[url:197k8v9e]http://antrodimitch.wordpress.com[/url:197k8v9e]

Membro del fan club di Ippo_
Thebear
Messaggi: 311
Iscritto il: 13 feb 2008, 16:23
Località: Torino

Messaggio da Thebear »

Quindi ho inteso irriducibile nel modo corretto? Beh, allora adesso penso alla conclusione!
Edoardo
Thebear
Messaggi: 311
Iscritto il: 13 feb 2008, 16:23
Località: Torino

Messaggio da Thebear »

Potrebbe funzionare un ragionamento per induzione partendo dall'irriducibilità di $ x+4 $, $ x^2+4 $ e $ x^3+4 $ utilizzando come passo induttivo 4?
Edoardo
Giulius
Messaggi: 58
Iscritto il: 02 apr 2009, 21:49
Località: Milano

Messaggio da Giulius »

Dimostriamo l'Hint di Jordan:
le radici di $ x^n=-4 $ se riportate sul piano di Gauss giacciono sulla circonferenza di raggio $ 4^1^/^n $, ossia il modulo delle radici è $ 2^2^/^n $.
Supponiamo $ p(x) $ e $ q(x) $ in $ Q[x] $ tali che $ p(x)q(x)=x^n+4 $. Pongo $ k=deg(p) $ (evidentemente $ deg(q)=n-k $). Inoltre per il lemma di Gauss possiamo supporre $ p(x) $ e $ q(x) $ in $ Z[x] $. Detti $ z_1 $ e $ z_2 $ i termini noti di $ p(x) $ e $ q(x) $ abbiamo che:
$ z_1z_2=4, z_1=2^2^k^/^n, z_2=2^2^(^n^-^k^)^/^n $, da cui $ z_1=z_2=2 $ (poichè $ z_1 $ e $ z_2 $ devono essere interi), da cui otteniamo (ponendo $ 2k/n=2(n-k)/n=1 $) $ n=2k $ In questo modo abbiamo però solamente mostrato che $ n $ è pari. Mostriamo che l'ipotesi $ k $ dispari è assurda:
Riscriviamo $ x^n+4=x^2^k+4 $, se supponessimo $ k $ dispari allora anche $ n-k $ sarebbe dispari e quindi sia $ p(x) $ che $ q(x) $ ammetterebbero almeno una radice reale. Ma il polinomio $ x^2^k+4 $ non ha alcuna radice reale, da cui l'assurdo.
Inoltre come postato sopra da TheBear il caso $ 4|n $ è sistemato.
Ultima modifica di Giulius il 20 apr 2009, 16:49, modificato 1 volta in totale.
Aboliamo il latino nei licei scientifici!
Giulius
Messaggi: 58
Iscritto il: 02 apr 2009, 21:49
Località: Milano

Messaggio da Giulius »

Visto che siamo in vena di polinomi e riducibilità....
Problema 7: Trovare gli interi $ k $ tali che il polinomio $ P(x)=x^5+x^4+x^3+kx^2+x+1 $ sia riducibile in $ Z[x] $.
Aboliamo il latino nei licei scientifici!
Avatar utente
jordan
Messaggi: 3988
Iscritto il: 02 feb 2007, 21:19
Località: Pescara
Contatta:

Messaggio da jordan »

Soluzione problema 7. Sia $ \displaystyle P(x)=\sum_{i=0}^5{x^i}+(k-1)x^2 \in \mathbb{Z}[x] $; se esistono $ P_1(x),P_2(x) \in \mathbb{Z}[x] $ non costanti tali che $ P(x)=P_1(x)P_2(x) $ allora wlog $ 0<deg(P_1(x))< deg(P_2(x))<5 $. Allora wlog $ 0<deg(P_1(x))<3 $. Se $ deg(P_1(x))=1 $ allora $ \exists \alpha \in \mathbb{Z} $ tale che $ |\alpha|=1 $ e $ P(\alpha)=0 $. Ma $ P(1)=P(-1)=0 $ ha soluzione sse $ k \in \{1,-5\} $. Resta solo il caso $ deg(P_1(x))=2 $: wlog esistono $ (a,b,c) \in \mathbb{Z}^3 $ ed $ \epsilon \in \{-1,1\} $ tali che $ P_1(x)=x^2+ax+\epsilon \text{ e } P_2(x)=x^3+bx^2+cx +\epsilon $. Ma la condizione di riducibilità è allora equivalente al sistema:
i) $ a+b=1 $
ii) $ c+ab + \epsilon=1 $
iii) $ a+c= \epsilon $.
iv) $ k=ac+\epsilon(b+1) $
Se $ \epsilon=1 $ allora $ b=c=1-a $ cosicchè $ (1-a)(a+1)=0 $. Allora $ k=1 $ in entrambi i casi. Se $ \epsilon=-1 $ allora $ b=c+2=1-a $ per cui $ (c+1)(c+2)-c+2=(c+1)^2+3=0 $, assurdo.
Conclusione: $ P(x) $ è riducibile in$ \mathbb{Z}[x] $ se e solo se $ k \in \{1,-5\} $.
The only goal of science is the honor of the human spirit.
Avatar utente
jordan
Messaggi: 3988
Iscritto il: 02 feb 2007, 21:19
Località: Pescara
Contatta:

Messaggio da jordan »

Problema 8. (Own).
Sia $ \{\sigma(1), \sigma(3), \sigma(5),..., \sigma(2009)\} $ una permutazione di $ \{1,3,5,...,2009\} $.
Sapendo che $ \displaystyle P(x)=\sum_{i=0}^{1004}{(-1)^i\sigma(2i+1)x^{2i}} $ non è monico, mostare che se esiste un $ \alpha \in \mathbb{R} $ tale che $ P(\alpha)=0 $ allora $ \alpha+29>0 $.
The only goal of science is the honor of the human spirit.
Avatar utente
julio14
Messaggi: 1208
Iscritto il: 11 dic 2006, 18:52
Località: Berlino

Messaggio da julio14 »

Poiché $ $P(\alpha)=0\Leftrightarrow P(-\alpha)=0 $ la tesi si può riformulare come $ $|\alpha|\ge 29\Rightarrow P(\alpha)\neq 0 $

$ $\sigma (2009)x^{2008}-\sigma(2007)x^{2006}+...-\sigma(3)x^2+\sigma(1) $
raggruppiamo a coppie:
$ $x^{2006}(\sigma(2009)x^2-\sigma(2007))+x^{2002}(\sigma(2005)x^2-\sigma(2003))+...+x^2(\sigma(5)x^2-\sigma(3))x+\sigma(1) $
Poiché per ipotesi $ $\sigma(2009)\neq 1 $ e quindi $ $\sigma(2009)\ge 3 $ per $ $x\ge 29 $ abbiamo che
$ $x^{2006}(\sigma(2009)x^2-\sigma(2007))\ge 29^{2006}(3\cdot 29^2-2009)=29^{2006}\cdot 514 $
Ugualmente tutte le altre coppie meno al più una sono positive. La minima coppia negativa possibile è $ $x^{2002}(x^2-2009) $, ma già $ $x^{2006}(\sigma(2009)x^2-\sigma(2007))+x^{2002}(\sigma(2005)x^2-\sigma(2003))>x^{2006}(3x^2-2009)+x^{2002}(x^2-2009)>0 $
che per $ $|x|>29 $ è vera per considerazioni sugli ordini di grandezza o facilmente con le derivate

Problema 9
boh non ho nulla di interessante. Volevo solo risollevare un po' il topic che rischiava la putrefazione. Qualcuno ha un bel problema? (jordan escluso per manifesto tentativo di impossessarsi del topic, come d'altronde del resto del forum invadendolo di problemi :D )
Anche se lo scrive qualche novellino che abbassa un po' il livello della staffetta non è una tragedia, eh.
Simo_the_wolf
Moderatore
Messaggi: 1053
Iscritto il: 01 gen 1970, 01:00
Località: Pescara

Messaggio da Simo_the_wolf »

Beh andando sul classico (facilotto, come richiesto da julio14)

Problema 9

a) Sia dato $ a \in \mathbb{Z} $ nonnullo. Sapendo che $ P(i)=a^i $ se $ i=0,..,n $ e che il grado di $ P(x) $ è minore o uguale a $ n $, sai dirmi quanto è $ P(n+1) $?

b) Dato $ a \in \mathbb{Z} $ con $ |a|>1 $, esiste un polinomio a coefficienti interi non costante di grado minore o uguale a $ n $ tale che $ P(i) $ è una potenza di $ a $ per $ i=0,..,n $ (numeri distinti, potenze distinte)?
pak-man
Messaggi: 313
Iscritto il: 07 giu 2008, 18:19

Messaggio da pak-man »

Se non si trattasse di polinomi, ponendo $ $q(x)=\frac{p(x)}{a^x}-1=mx(x-1)\cdots(x-n) $ (con m non nullo) si trova che $ p(n+1)=a^{n+1}(m\cdot(n+1)!+1) $. Ma $ ~p(x) $ è un polinomio... :roll:
spugna
Messaggi: 421
Iscritto il: 19 mar 2009, 22:18
Località: Forlì

Messaggio da spugna »

Simo_the_wolf ha scritto:Problema 9

a) Sia dato $ a \in \mathbb{Z} $ nonnullo. Sapendo che $ P(i)=a^i $ se $ i=0,..,n $ e che il grado di $ P(x) $ è minore o uguale a $ n $, sai dirmi quanto è $ P(n+1) $?
Si potrebbe pensare alla regola $ P(i) = a^k $ con $ k = i $ $ mod $ $ (n + 1) $. In effetti con $ i \leq n $ si ha $ i = k $, da cui $ P(i) = a^k = a^i $. Con $ i = n+1 $ invece si ha $ k = (n + 1) $ $ mod $ $ (n + 1) = 0 \Rightarrow P(n+1) = a^k = a^0 = 1 $ $ \forall a \neq 0 $.

Per il problema (b) però non ho idee (per ora)
"Bene, ora dobbiamo massimizzare [tex]\dfrac{x}{(x+100)^2}[/tex]: come possiamo farlo senza le derivate? Beh insomma, in zero fa zero... a $+\infty$ tende a zero... e il massimo? Potrebbe essere, che so, in $10^{24}$? Chiaramente no... E in $10^{-3}$? Nemmeno... Insomma, nella frazione c'è solo il numero $100$, quindi dove volete che sia il massimo se non in $x=100$..?" (da leggere con risatine perfide e irrisorie in corrispondenza dei puntini di sospensione)

Maledetti fisici! (cit.)
stefanos
Messaggi: 229
Iscritto il: 02 giu 2008, 13:23
Località: Roma
Contatta:

Messaggio da stefanos »

E` passato un po' di tempo... Propongo una mia soluzione.
a. Sia $ $q(x) = p(x) - \sum_{k=0}^n a^k \prod_{j=0, j\neq k}^n \frac{x-j}{k-j}$ $.

Notiamo che il grado di questo polinomio e` al piu` $ $n$ $, perche' il grado di $ $p(x)$ $ e` minore o uguale a $ $n$ $ e il grado degli altri termini e` pure $ $n$ $. Tuttavia, questo polinomio si annulla per tutti gli $ ${1, a, a^2, \cdots, a^n}$ $, quindi ha $ $n+1$ $ radici. Allora deve essere identicamente nullo, cioe`
$ $p(x) = \sum_{k=0}^n a^k \prod_{j=0, j\neq k}^n \frac{x-j}{k-j},$ $
e quindi si trova un certo valore per $ $p(n+1)$ $, piuttosto brutto ;)

b. Se ho capito bene il testo mi basta fare in modo che $ $\prod_{j=0, j\neq k}^n(k-j) | a^k$ $, quindi mi sara` sufficiente porre $ $a = \prod_{k=0}^n \prod_{j=0, j\neq k}^n(k-j)$ $. Mi sembra che questa roba si scriva meglio con un po' di fattoriali... ma quel che importa e` che si possa fare.

C'e` qualche errore?
Physics is like sex. Sure, it may give some practical results, but that's not why we do it.
Edriv: c=c+2; "tu sarai ricordato come `colui che ha convertito edriv alla fisica' ;)"
[quote="Tibor Gallai"]Alla fine sono macchine di Turing pure loro, solo un po' meno deterministiche di noi.[/quote]
Avatar utente
federiko97
Messaggi: 44
Iscritto il: 03 nov 2008, 20:36
Località: Roma

Messaggio da federiko97 »

Uhm, forse c'è un modo per rendere più decorosa l'idea di stefanos:

pongo $ \displaystyle q_i(x)=\frac{x(x-1)\dots (x-i+1)}{i!} $ (e $ q_0=1 $, in caso non fosse chiaro)

Preso $ k\le n $ noto che $ \displaystyle a^k=\sum_{i=0}^k {k\choose i}(a-1)^i=\sum_{i=0}^n q_i(k)(a-1)^i $

Quindi $ \displaystyle P(x)=\sum_{i=0}^n q_i(x)(a-1)^i $ da cui $ \displaystyle P(n+1)=\sum_{i=0}^n q_i(n+1)(a-1)^i=\sum_{i=0}^n {n+1\choose i}(a-1)^i= $$ \displaystyle \sum_{i=0}^{n+1} {n+1\choose i}(a-1)^i-(a-1)^{n+1}=a^{n+1}-(a-1)^{n+1} $
Io credo che alcune entità superiori, pur non avendo odore, possano esistere. Esse influenzano le nostre vite in maniera che nessuno scienziato può comprendere.
Rispondi